Inverse Laplace transform (Initial Value Problem)

Click For Summary
The discussion revolves around finding the inverse Laplace transform of the function Y(s) = 1 / [(s-1)^2 + 1]^2 to solve a differential equation. The original poster struggled with using the translation theorem and partial fractions, feeling stuck on the squared denominator. Other participants encouraged sharing more details about their attempts to provide better assistance. Ultimately, the poster realized that applying a formula from Laplace transform tables was the correct approach, rather than using partial fractions. The conversation highlights the importance of recognizing when to apply specific mathematical formulas in solving complex problems.
aero_zeppelin
Messages
85
Reaction score
0

Homework Statement



I'm stuck trying to find out the inverse Laplace of F(s) to get y(t) (the solution for the differential equation):

Y(s) = 1 / [ (s-1)^2 + 1 ]^2



The Attempt at a Solution



I tried using a translation theorem and then apply the sine formula, but the denominator is still all squared. I also tried partial fractions to expand Y(s) but I didn't get it right...

Any suggestions please?
 
Physics news on Phys.org
aero_zeppelin said:

Homework Statement



I'm stuck trying to find out the inverse Laplace of F(s) to get y(t) (the solution for the differential equation):

Y(s) = 1 / [ (s-1)^2 + 1 ]^2



The Attempt at a Solution



I tried using a translation theorem and then apply the sine formula, but the denominator is still all squared. I also tried partial fractions to expand Y(s) but I didn't get it right...

Any suggestions please?

Yes: show us what you did and what you got. How can we possibly help if we have no idea of what your problem is?
 
[(s-1)+1]^2=[(s-1+i)(s-1-i)]^2

Therefore, \left[\frac{1}{ (s-1-i)(s-1+i) }\right]^2=\left[\frac{1}{2i}\left(\frac{1}{s-1-i}-\frac{1}{s-1+i} \right ) \right ]^2

Can you continue it from there?
 
  • Like
Likes 1 person
Ray Vickson said:
Yes: show us what you did and what you got. How can we possibly help if we have no idea of what your problem is?


My friend, that's the thing, I can't get past that. I stated the problem clearly: "I'm stuck trying to find out the INVERSE LAPLACE of the given Y(s) to get y(t)". There's no point in showing what I did before, it's not needed for what's after the equation I posted.

Anyway, I got it. It was just applying one of the formulas from the Laplace transform tables. I thought partial fractions or something else had to be done.

Thanks for your help
 
Question: A clock's minute hand has length 4 and its hour hand has length 3. What is the distance between the tips at the moment when it is increasing most rapidly?(Putnam Exam Question) Answer: Making assumption that both the hands moves at constant angular velocities, the answer is ## \sqrt{7} .## But don't you think this assumption is somewhat doubtful and wrong?

Similar threads

  • · Replies 10 ·
Replies
10
Views
2K
  • · Replies 2 ·
Replies
2
Views
2K
  • · Replies 4 ·
Replies
4
Views
3K
Replies
2
Views
2K
  • · Replies 4 ·
Replies
4
Views
3K
Replies
9
Views
2K
  • · Replies 8 ·
Replies
8
Views
2K
Replies
5
Views
2K
  • · Replies 7 ·
Replies
7
Views
2K
  • · Replies 1 ·
Replies
1
Views
1K